Find a partition of nodes into two sets $V_1$ and $V_2$ such that these two sets cover most edges












0












$begingroup$


Given an undirected connected graph $G$. Let $V$ be its node set.
The problem is to find a partition of $V$ ($V = V_1 +V_2$ and $V_1$, $V_2$ are disjoint) such that $|Cover(V_1)| + |Cover(V_2)|$ is maximized, where $Cover(V_i)$ is the set of edges incident to at least one node in $V_i$ (that is, the edges covered by $V_i$).



I wonder if the following hypothesis is correct:
$|Cover(V_1)| + |Cover(V_2)|$ is maximized if and only if one of $V_1$ and $V_2$ is a minimum vertex cover of the graph $G$.










share|cite|improve this question









$endgroup$

















    0












    $begingroup$


    Given an undirected connected graph $G$. Let $V$ be its node set.
    The problem is to find a partition of $V$ ($V = V_1 +V_2$ and $V_1$, $V_2$ are disjoint) such that $|Cover(V_1)| + |Cover(V_2)|$ is maximized, where $Cover(V_i)$ is the set of edges incident to at least one node in $V_i$ (that is, the edges covered by $V_i$).



    I wonder if the following hypothesis is correct:
    $|Cover(V_1)| + |Cover(V_2)|$ is maximized if and only if one of $V_1$ and $V_2$ is a minimum vertex cover of the graph $G$.










    share|cite|improve this question









    $endgroup$















      0












      0








      0





      $begingroup$


      Given an undirected connected graph $G$. Let $V$ be its node set.
      The problem is to find a partition of $V$ ($V = V_1 +V_2$ and $V_1$, $V_2$ are disjoint) such that $|Cover(V_1)| + |Cover(V_2)|$ is maximized, where $Cover(V_i)$ is the set of edges incident to at least one node in $V_i$ (that is, the edges covered by $V_i$).



      I wonder if the following hypothesis is correct:
      $|Cover(V_1)| + |Cover(V_2)|$ is maximized if and only if one of $V_1$ and $V_2$ is a minimum vertex cover of the graph $G$.










      share|cite|improve this question









      $endgroup$




      Given an undirected connected graph $G$. Let $V$ be its node set.
      The problem is to find a partition of $V$ ($V = V_1 +V_2$ and $V_1$, $V_2$ are disjoint) such that $|Cover(V_1)| + |Cover(V_2)|$ is maximized, where $Cover(V_i)$ is the set of edges incident to at least one node in $V_i$ (that is, the edges covered by $V_i$).



      I wonder if the following hypothesis is correct:
      $|Cover(V_1)| + |Cover(V_2)|$ is maximized if and only if one of $V_1$ and $V_2$ is a minimum vertex cover of the graph $G$.







      combinatorics graph-theory computational-complexity






      share|cite|improve this question













      share|cite|improve this question











      share|cite|improve this question




      share|cite|improve this question










      asked Nov 28 '18 at 17:04









      ParadoxParadox

      889




      889






















          1 Answer
          1






          active

          oldest

          votes


















          1












          $begingroup$

          Your hypothesis is incorrect. Let $G = G_1 cup G_2 cup ldots G_m$ where each $G_i$ is a complete bipartite graph where the left side $L_{i}$ of $G_i$ has half as many vertices as the right side $R_{i}$ of $G_i$. The $G_i$'s are vertex-disjoint; so $G$ is $m$ connected components where $G$ on the $i$-th component is the graph $G_i$.



          The unique minimum vertex cover of $G$ is $L_1 cup ldots L_m$.



          However, construct $V_1$ and $V_2$ as follows: for each $i=1,ldots, m$ pick an arbitrary side of $G_i$ and put it in $V_1$ and put the other side into $V_2$. Then both $V_1$ and $V_2$ are disjoint and each cover all the edges, so $|Cov(V_1)| + |Cov(V_2)|$ is definitely maximized. But neither $V_1$ nor $V_2$ has to be a minimum vertex-cover--indeed $V_1$ can have the left side of some but not all of the $G_i$s while $V_2$ can have simultaneously the left side of some but not all of the $G_i$s.



          If you insist on connectedness put an edge between $L_i$ and $L_{i+1}$ for each $i$. Then $L_1 cup ldots L_m$ is still the unique minimum vertex cover. But consider $V_1 = cup_{i odd} L_i + cup_{i even} R_i$ while $V_2$ is $V_1$'s complement.






          share|cite|improve this answer











          $endgroup$













            Your Answer





            StackExchange.ifUsing("editor", function () {
            return StackExchange.using("mathjaxEditing", function () {
            StackExchange.MarkdownEditor.creationCallbacks.add(function (editor, postfix) {
            StackExchange.mathjaxEditing.prepareWmdForMathJax(editor, postfix, [["$", "$"], ["\\(","\\)"]]);
            });
            });
            }, "mathjax-editing");

            StackExchange.ready(function() {
            var channelOptions = {
            tags: "".split(" "),
            id: "69"
            };
            initTagRenderer("".split(" "), "".split(" "), channelOptions);

            StackExchange.using("externalEditor", function() {
            // Have to fire editor after snippets, if snippets enabled
            if (StackExchange.settings.snippets.snippetsEnabled) {
            StackExchange.using("snippets", function() {
            createEditor();
            });
            }
            else {
            createEditor();
            }
            });

            function createEditor() {
            StackExchange.prepareEditor({
            heartbeatType: 'answer',
            autoActivateHeartbeat: false,
            convertImagesToLinks: true,
            noModals: true,
            showLowRepImageUploadWarning: true,
            reputationToPostImages: 10,
            bindNavPrevention: true,
            postfix: "",
            imageUploader: {
            brandingHtml: "Powered by u003ca class="icon-imgur-white" href="https://imgur.com/"u003eu003c/au003e",
            contentPolicyHtml: "User contributions licensed under u003ca href="https://creativecommons.org/licenses/by-sa/3.0/"u003ecc by-sa 3.0 with attribution requiredu003c/au003e u003ca href="https://stackoverflow.com/legal/content-policy"u003e(content policy)u003c/au003e",
            allowUrls: true
            },
            noCode: true, onDemand: true,
            discardSelector: ".discard-answer"
            ,immediatelyShowMarkdownHelp:true
            });


            }
            });














            draft saved

            draft discarded


















            StackExchange.ready(
            function () {
            StackExchange.openid.initPostLogin('.new-post-login', 'https%3a%2f%2fmath.stackexchange.com%2fquestions%2f3017398%2ffind-a-partition-of-nodes-into-two-sets-v-1-and-v-2-such-that-these-two-sets%23new-answer', 'question_page');
            }
            );

            Post as a guest















            Required, but never shown

























            1 Answer
            1






            active

            oldest

            votes








            1 Answer
            1






            active

            oldest

            votes









            active

            oldest

            votes






            active

            oldest

            votes









            1












            $begingroup$

            Your hypothesis is incorrect. Let $G = G_1 cup G_2 cup ldots G_m$ where each $G_i$ is a complete bipartite graph where the left side $L_{i}$ of $G_i$ has half as many vertices as the right side $R_{i}$ of $G_i$. The $G_i$'s are vertex-disjoint; so $G$ is $m$ connected components where $G$ on the $i$-th component is the graph $G_i$.



            The unique minimum vertex cover of $G$ is $L_1 cup ldots L_m$.



            However, construct $V_1$ and $V_2$ as follows: for each $i=1,ldots, m$ pick an arbitrary side of $G_i$ and put it in $V_1$ and put the other side into $V_2$. Then both $V_1$ and $V_2$ are disjoint and each cover all the edges, so $|Cov(V_1)| + |Cov(V_2)|$ is definitely maximized. But neither $V_1$ nor $V_2$ has to be a minimum vertex-cover--indeed $V_1$ can have the left side of some but not all of the $G_i$s while $V_2$ can have simultaneously the left side of some but not all of the $G_i$s.



            If you insist on connectedness put an edge between $L_i$ and $L_{i+1}$ for each $i$. Then $L_1 cup ldots L_m$ is still the unique minimum vertex cover. But consider $V_1 = cup_{i odd} L_i + cup_{i even} R_i$ while $V_2$ is $V_1$'s complement.






            share|cite|improve this answer











            $endgroup$


















              1












              $begingroup$

              Your hypothesis is incorrect. Let $G = G_1 cup G_2 cup ldots G_m$ where each $G_i$ is a complete bipartite graph where the left side $L_{i}$ of $G_i$ has half as many vertices as the right side $R_{i}$ of $G_i$. The $G_i$'s are vertex-disjoint; so $G$ is $m$ connected components where $G$ on the $i$-th component is the graph $G_i$.



              The unique minimum vertex cover of $G$ is $L_1 cup ldots L_m$.



              However, construct $V_1$ and $V_2$ as follows: for each $i=1,ldots, m$ pick an arbitrary side of $G_i$ and put it in $V_1$ and put the other side into $V_2$. Then both $V_1$ and $V_2$ are disjoint and each cover all the edges, so $|Cov(V_1)| + |Cov(V_2)|$ is definitely maximized. But neither $V_1$ nor $V_2$ has to be a minimum vertex-cover--indeed $V_1$ can have the left side of some but not all of the $G_i$s while $V_2$ can have simultaneously the left side of some but not all of the $G_i$s.



              If you insist on connectedness put an edge between $L_i$ and $L_{i+1}$ for each $i$. Then $L_1 cup ldots L_m$ is still the unique minimum vertex cover. But consider $V_1 = cup_{i odd} L_i + cup_{i even} R_i$ while $V_2$ is $V_1$'s complement.






              share|cite|improve this answer











              $endgroup$
















                1












                1








                1





                $begingroup$

                Your hypothesis is incorrect. Let $G = G_1 cup G_2 cup ldots G_m$ where each $G_i$ is a complete bipartite graph where the left side $L_{i}$ of $G_i$ has half as many vertices as the right side $R_{i}$ of $G_i$. The $G_i$'s are vertex-disjoint; so $G$ is $m$ connected components where $G$ on the $i$-th component is the graph $G_i$.



                The unique minimum vertex cover of $G$ is $L_1 cup ldots L_m$.



                However, construct $V_1$ and $V_2$ as follows: for each $i=1,ldots, m$ pick an arbitrary side of $G_i$ and put it in $V_1$ and put the other side into $V_2$. Then both $V_1$ and $V_2$ are disjoint and each cover all the edges, so $|Cov(V_1)| + |Cov(V_2)|$ is definitely maximized. But neither $V_1$ nor $V_2$ has to be a minimum vertex-cover--indeed $V_1$ can have the left side of some but not all of the $G_i$s while $V_2$ can have simultaneously the left side of some but not all of the $G_i$s.



                If you insist on connectedness put an edge between $L_i$ and $L_{i+1}$ for each $i$. Then $L_1 cup ldots L_m$ is still the unique minimum vertex cover. But consider $V_1 = cup_{i odd} L_i + cup_{i even} R_i$ while $V_2$ is $V_1$'s complement.






                share|cite|improve this answer











                $endgroup$



                Your hypothesis is incorrect. Let $G = G_1 cup G_2 cup ldots G_m$ where each $G_i$ is a complete bipartite graph where the left side $L_{i}$ of $G_i$ has half as many vertices as the right side $R_{i}$ of $G_i$. The $G_i$'s are vertex-disjoint; so $G$ is $m$ connected components where $G$ on the $i$-th component is the graph $G_i$.



                The unique minimum vertex cover of $G$ is $L_1 cup ldots L_m$.



                However, construct $V_1$ and $V_2$ as follows: for each $i=1,ldots, m$ pick an arbitrary side of $G_i$ and put it in $V_1$ and put the other side into $V_2$. Then both $V_1$ and $V_2$ are disjoint and each cover all the edges, so $|Cov(V_1)| + |Cov(V_2)|$ is definitely maximized. But neither $V_1$ nor $V_2$ has to be a minimum vertex-cover--indeed $V_1$ can have the left side of some but not all of the $G_i$s while $V_2$ can have simultaneously the left side of some but not all of the $G_i$s.



                If you insist on connectedness put an edge between $L_i$ and $L_{i+1}$ for each $i$. Then $L_1 cup ldots L_m$ is still the unique minimum vertex cover. But consider $V_1 = cup_{i odd} L_i + cup_{i even} R_i$ while $V_2$ is $V_1$'s complement.







                share|cite|improve this answer














                share|cite|improve this answer



                share|cite|improve this answer








                edited Dec 2 '18 at 1:39

























                answered Nov 28 '18 at 19:29









                MikeMike

                3,785411




                3,785411






























                    draft saved

                    draft discarded




















































                    Thanks for contributing an answer to Mathematics Stack Exchange!


                    • Please be sure to answer the question. Provide details and share your research!

                    But avoid



                    • Asking for help, clarification, or responding to other answers.

                    • Making statements based on opinion; back them up with references or personal experience.


                    Use MathJax to format equations. MathJax reference.


                    To learn more, see our tips on writing great answers.




                    draft saved


                    draft discarded














                    StackExchange.ready(
                    function () {
                    StackExchange.openid.initPostLogin('.new-post-login', 'https%3a%2f%2fmath.stackexchange.com%2fquestions%2f3017398%2ffind-a-partition-of-nodes-into-two-sets-v-1-and-v-2-such-that-these-two-sets%23new-answer', 'question_page');
                    }
                    );

                    Post as a guest















                    Required, but never shown





















































                    Required, but never shown














                    Required, but never shown












                    Required, but never shown







                    Required, but never shown

































                    Required, but never shown














                    Required, but never shown












                    Required, but never shown







                    Required, but never shown







                    Popular posts from this blog

                    Biblatex bibliography style without URLs when DOI exists (in Overleaf with Zotero bibliography)

                    ComboBox Display Member on multiple fields

                    Is it possible to collect Nectar points via Trainline?